Mathcenter Forum  

Go Back   Mathcenter Forum > คณิตศาสตร์โอลิมปิก และอุดมศึกษา > อสมการ
สมัครสมาชิก คู่มือการใช้ รายชื่อสมาชิก ปฏิทิน ข้อความวันนี้

ตั้งหัวข้อใหม่ Reply
 
เครื่องมือของหัวข้อ ค้นหาในหัวข้อนี้
  #1  
Old 22 กรกฎาคม 2011, 21:24
BLACK-Dragon's Avatar
BLACK-Dragon BLACK-Dragon ไม่อยู่ในระบบ
กระบี่ประสานใจ
 
วันที่สมัครสมาชิก: 04 พฤศจิกายน 2010
ข้อความ: 719
BLACK-Dragon is on a distinguished road
Default อสมการ

Show that for all positive a,b,c

$$(\dfrac{a+2b}{a+2c})^3+(\dfrac{b+2c}{b+2a})^3+(\dfrac{c+2a}{c+2b})^3 \ge 3$$

ตอบพร้อมอ้างอิงข้อความนี้
  #2  
Old 22 กรกฎาคม 2011, 22:32
nooonuii nooonuii ไม่อยู่ในระบบ
ผู้พิทักษ์กฎทั่วไป
 
วันที่สมัครสมาชิก: 25 พฤษภาคม 2001
ข้อความ: 6,408
nooonuii is on a distinguished road
Default

อ้างอิง:
ข้อความเดิมเขียนโดยคุณ BLACK-Dragon View Post
Show that for all positive a,b,c

$$(\dfrac{a+2b}{a+2c})^3+(\dfrac{b+2c}{b+2a})^3+(\dfrac{c+2a}{c+2b})^3 \ge 3$$
อันนี้ก็จริง

$$(\dfrac{a+3b}{a+3c})^3+(\dfrac{b+3c}{b+3a})^3+(\dfrac{c+3a}{c+3b})^3 \ge 3$$
__________________
site:mathcenter.net คำค้น
ตอบพร้อมอ้างอิงข้อความนี้
  #3  
Old 22 กรกฎาคม 2011, 22:33
BLACK-Dragon's Avatar
BLACK-Dragon BLACK-Dragon ไม่อยู่ในระบบ
กระบี่ประสานใจ
 
วันที่สมัครสมาชิก: 04 พฤศจิกายน 2010
ข้อความ: 719
BLACK-Dragon is on a distinguished road
Default

อ้างอิง:
ข้อความเดิมเขียนโดยคุณ nooonuii View Post
อันนี้ก็จริง

$$(\dfrac{a+3b}{a+3c})^3+(\dfrac{b+3c}{b+3a})^3+(\dfrac{c+3a}{c+3b})^3 \ge 3$$
หมายถึงอะไรหรอครับ แล้วผมพลาดตรงไหนหรอครับ
ตอบพร้อมอ้างอิงข้อความนี้
  #4  
Old 22 กรกฎาคม 2011, 22:38
Keehlzver's Avatar
Keehlzver Keehlzver ไม่อยู่ในระบบ
บัณฑิตฟ้า
 
วันที่สมัครสมาชิก: 26 มกราคม 2009
ข้อความ: 533
Keehlzver is on a distinguished road
Default

จะเข้ามาบอกว่าวิธีสวยมากๆเลยครับ
__________________
"ชั่วโมงหน้าต้องดีกว่าเดิม!"
ตอบพร้อมอ้างอิงข้อความนี้
  #5  
Old 22 กรกฎาคม 2011, 22:44
nooonuii nooonuii ไม่อยู่ในระบบ
ผู้พิทักษ์กฎทั่วไป
 
วันที่สมัครสมาชิก: 25 พฤษภาคม 2001
ข้อความ: 6,408
nooonuii is on a distinguished road
Default

อ้างอิง:
ข้อความเดิมเขียนโดยคุณ BLACK-Dragon View Post
หมายถึงอะไรหรอครับ แล้วผมพลาดตรงไหนหรอครับ
ดูแล้วไม่มีที่ผิดครับ อสมการที่ผมให้มาก็จริงครับ ผมคิดโดยใช้ power mean + Cauchy-Schwarz
__________________
site:mathcenter.net คำค้น
ตอบพร้อมอ้างอิงข้อความนี้
  #6  
Old 22 กรกฎาคม 2011, 22:51
BLACK-Dragon's Avatar
BLACK-Dragon BLACK-Dragon ไม่อยู่ในระบบ
กระบี่ประสานใจ
 
วันที่สมัครสมาชิก: 04 พฤศจิกายน 2010
ข้อความ: 719
BLACK-Dragon is on a distinguished road
Default

อ้างอิง:
ข้อความเดิมเขียนโดยคุณ Keehlzver View Post
จะเข้ามาบอกว่าวิธีสวยมากๆเลยครับ
ขอบคุณครับ(เหมือนผิดๆ ไงไม่รู้ )

มีอีก 2 ข้อมาฝากครับ

1. Let a,b,c be positive reals.Prove that

$$\dfrac{a}{\sqrt{a^2+8bc}}+\dfrac{b}{\sqrt{b^2+8ca}}+\dfrac{c}{\sqrt{c^2+8ab}} \ge 1$$

2.Let a,b,c be positive reals a+b+c=1.Show that

$$\dfrac{a}{\sqrt{b+c}}+\dfrac{b}{\sqrt{c+a}}+\dfrac{c}{\sqrt{a+b}}\ge \sqrt{\dfrac{3}{2}}$$

โจทย์แนวนี้ที่มีติดรูทผมไม่ค่อยได้เลยอ่ะครับ พอเจอความรู้สึกมันก็คิดไปแล้วว่าทำไม่ได้ เซ็งมากครับ

ปล.คุณ Keehlzver ไป IMO มาใช่ไหมครับเนี่ย
ตอบพร้อมอ้างอิงข้อความนี้
  #7  
Old 22 กรกฎาคม 2011, 23:38
Keehlzver's Avatar
Keehlzver Keehlzver ไม่อยู่ในระบบ
บัณฑิตฟ้า
 
วันที่สมัครสมาชิก: 26 มกราคม 2009
ข้อความ: 533
Keehlzver is on a distinguished road
Default

ไม่ผิดครับ ไอเดียที่ทำมาก็สวยแล้วด้วย
สองข้อ ข้อแรกใช้ Holder แล้วก็อสมการพื้นฐาน ส่วนข้อสองถ้าผมจำไม่ผิดก็ Holder อีกนั่นแหละไม่ก็ใช้ความเป็น Convex Concave (เชิญพี่ Nooonuii ตามระเบียบเช่นเดิม )

(ปล.ผมเคยไปมาครับ IMO แต่เป็น I may out! แปลว่าฉันอาจสอบตกครับ จริงๆเว็บบอร์ดที่นี่ระดับตัวแทนเข้ามาตอบน้อยมากเลยครับ ไม่ค่อยว่างกันหรอกพวกนี้)

(ปล.2 เพิ่งเริ่มฝึกอสมการใช่หรือเปล่า ลองทำ Hojoo Lee ดูรึยังครับ ทำหมดนั่นรับรองเก่งขึ้นแน่ๆ ผมเป็นกำลังใจให้ครับ )
__________________
"ชั่วโมงหน้าต้องดีกว่าเดิม!"
ตอบพร้อมอ้างอิงข้อความนี้
  #8  
Old 22 กรกฎาคม 2011, 23:40
Real Matrik's Avatar
Real Matrik Real Matrik ไม่อยู่ในระบบ
บัณฑิตฟ้า
 
วันที่สมัครสมาชิก: 14 พฤษภาคม 2011
ข้อความ: 386
Real Matrik is on a distinguished road
Default

ข้อ 2



22 กรกฎาคม 2011 23:45 : ข้อความนี้ถูกแก้ไขแล้ว 1 ครั้ง, ครั้งล่าสุดโดยคุณ Real Matrik
ตอบพร้อมอ้างอิงข้อความนี้
  #9  
Old 23 กรกฎาคม 2011, 08:11
จูกัดเหลียง's Avatar
จูกัดเหลียง จูกัดเหลียง ไม่อยู่ในระบบ
ลมปราณไร้สภาพ
 
วันที่สมัครสมาชิก: 21 กุมภาพันธ์ 2011
ข้อความ: 1,234
จูกัดเหลียง is on a distinguished road
Default

อ้างอิง:
ข้อความเดิมเขียนโดยคุณ BLACK-Dragon View Post

2.Let a,b,c be positive reals a+b+c=1.Show that

$$\dfrac{a}{\sqrt{b+c}}+\dfrac{b}{\sqrt{c+a}}+\dfrac{c}{\sqrt{a+b}}\ge \sqrt{\dfrac{3}{2}}$$
มั่วๆไปก่อนนะครับ
โดยอสมการของ Chebyshev's WLOG $a\ge b\ge c$
จะได้ว่า $$\frac{a}{ \sqrt{b+c}}+\frac{b}{ \sqrt{c+a}}+\frac{c}{\sqrt{a+b}}\ge \frac{1}{3}(a+b+c)(\frac{1}{ \sqrt{a+b}}+\frac{1}{ \sqrt{b+c}}+\frac{1}{ \sqrt{c+a}})=\frac{1}{3}(\frac{1}{ \sqrt{a+b}}+\frac{1}{ \sqrt{b+c}}+\frac{1}{ \sqrt{c+a}})$$
นั่นคือ ต้องการพิสูจน์ว่า $$(\frac{1}{\sqrt{a+b}}+\frac{1}{\sqrt{b+c}}+\frac{1}{\sqrt{c+a}})\ge 3\sqrt{\frac{3}{2}}$$
เเต่ $$(\frac{1}{\sqrt{a+b}}+\frac{1}{\sqrt{b+c}}+\frac{1}{\sqrt{c+a}})\ge 3\sqrt[3]{\frac{1}{\sqrt{(a+b)(b+c)(c+a)}}}$$
จึงเพียงพอที่จะพิสูจน์ว่า $$\frac{1}{\sqrt[6]{(a+b)(b+c)(c+a)}}\ge \sqrt{\frac{3}{2}}$$
$$\Leftrightarrow \sqrt[3]{(a+b)(b+c)(c+a)}\leq \frac{2}{3}=\frac{2}{3}(a+b+c)$$
$$\Leftrightarrow 3\sqrt[3]{(a+b)(b+c)(c+a)}\leq (a+b)+(b+c)+(c+a)$$
ซึ่งเป็นจริงโดย A.M.-G.M.

ว่างๆ เอามาลงอีกนะครับ อยากทำ 555+
__________________
Vouloir c'est pouvoir

23 กรกฎาคม 2011 08:25 : ข้อความนี้ถูกแก้ไขแล้ว 2 ครั้ง, ครั้งล่าสุดโดยคุณ จูกัดเหลียง
ตอบพร้อมอ้างอิงข้อความนี้
  #10  
Old 23 กรกฎาคม 2011, 08:42
BLACK-Dragon's Avatar
BLACK-Dragon BLACK-Dragon ไม่อยู่ในระบบ
กระบี่ประสานใจ
 
วันที่สมัครสมาชิก: 04 พฤศจิกายน 2010
ข้อความ: 719
BLACK-Dragon is on a distinguished road
Default

เดี๋ยวเองมาลงให้ก่อนละกันครับพี่จูเนียร์(เพราะช่วงนี้สอบกลางภาค)

3. Show that for all nozero reals a,b,c

$$\dfrac{a^2}{b^2}+\dfrac{b^2}{c^2}+\dfrac{c^2}{a^2} \ge \dfrac{a}{b}+\dfrac{b}{c}+\dfrac{c}{a}$$


23 กรกฎาคม 2011 15:37 : ข้อความนี้ถูกแก้ไขแล้ว 1 ครั้ง, ครั้งล่าสุดโดยคุณ BLACK-Dragon
ตอบพร้อมอ้างอิงข้อความนี้
  #11  
Old 23 กรกฎาคม 2011, 12:57
จูกัดเหลียง's Avatar
จูกัดเหลียง จูกัดเหลียง ไม่อยู่ในระบบ
ลมปราณไร้สภาพ
 
วันที่สมัครสมาชิก: 21 กุมภาพันธ์ 2011
ข้อความ: 1,234
จูกัดเหลียง is on a distinguished road
Default

ถ้าให้เดา คงใช้การนับเป็นเศษสส่วนใช่ไหมครับ
ผมว่ามันเเปลกๆนะ
__________________
Vouloir c'est pouvoir
ตอบพร้อมอ้างอิงข้อความนี้
  #12  
Old 23 กรกฎาคม 2011, 14:14
nooonuii nooonuii ไม่อยู่ในระบบ
ผู้พิทักษ์กฎทั่วไป
 
วันที่สมัครสมาชิก: 25 พฤษภาคม 2001
ข้อความ: 6,408
nooonuii is on a distinguished road
Default

อ้างอิง:
ข้อความเดิมเขียนโดยคุณ BLACK-Dragon View Post
เดี๋ยวเองมาลงให้ก่อนละกันครับพี่จูเนียร์(เพราะช่วงนี้สอบกลางภาค)

3. Show that for all nozero reals a,b,c

$$\dfrac{a^2}{b^2}+\dfrac{b^2}{c^2}+\dfrac{c^2}{a^2} \ge \dfrac{a}{b}+\dfrac{b}{c}+\dfrac{c}{a}$$

ให้ $x=\dfrac{a}{b},y=\dfrac{b}{c},z=\dfrac{c}{a}$ ได้ $xyz=1$

$x^2+y^2+z^2 \ge x+y+z$

$\displaystyle \dfrac{4}{3}x^2+\dfrac{1}{3}y^2+\dfrac{1}{3}z^2 \ge (x^{\frac{8}{3}}y^{\frac{2}{3}}z^{\frac{2}{3}})^{\frac{1}{2}}=x$

ตัวอื่นๆก็ทำเหมือนกันแล้วมาบวกกันให้หมด

$x^2+y^2+z^2 \ge x+y+z$
บวกกันแล้วได้ $2(x^2+y^2+z^2)\geq x+y+z$
__________________
site:mathcenter.net คำค้น
ตอบพร้อมอ้างอิงข้อความนี้
  #13  
Old 23 กรกฎาคม 2011, 14:24
จูกัดเหลียง's Avatar
จูกัดเหลียง จูกัดเหลียง ไม่อยู่ในระบบ
ลมปราณไร้สภาพ
 
วันที่สมัครสมาชิก: 21 กุมภาพันธ์ 2011
ข้อความ: 1,234
จูกัดเหลียง is on a distinguished road
Default

อ้างอิง:
ข้อความเดิมเขียนโดยคุณ nooonuii View Post
บวกกันแล้วได้ $2(x^2+y^2+z^2)\geq x+y+z$
ก็น้อง Black Dragon ลืมเลข 2 ของพจน์ทางขวาไม่ใช่เหรอครับ
__________________
Vouloir c'est pouvoir

23 กรกฎาคม 2011 14:51 : ข้อความนี้ถูกแก้ไขแล้ว 1 ครั้ง, ครั้งล่าสุดโดยคุณ จูกัดเหลียง
ตอบพร้อมอ้างอิงข้อความนี้
  #14  
Old 23 กรกฎาคม 2011, 15:36
BLACK-Dragon's Avatar
BLACK-Dragon BLACK-Dragon ไม่อยู่ในระบบ
กระบี่ประสานใจ
 
วันที่สมัครสมาชิก: 04 พฤศจิกายน 2010
ข้อความ: 719
BLACK-Dragon is on a distinguished road
Default

ขอบคุณครับที่ทักท้วง

ปล. Holder ใช่

$(a_1+a_2+...a_n)^{\theta_a}(b_1+b_2+...+b_n)^{\theta_b}...(z_1+z_2+...z_n)^{\theta_z} \ge a_1^{\theta_a}b_1^{\theta_b}...z_1^{\theta_z}+...+a_n^{\theta_a}b_n^{\theta_b}...z_n^{\theta_z}$

หรือเปล่าครับ
ตอบพร้อมอ้างอิงข้อความนี้
  #15  
Old 23 กรกฎาคม 2011, 17:52
จูกัดเหลียง's Avatar
จูกัดเหลียง จูกัดเหลียง ไม่อยู่ในระบบ
ลมปราณไร้สภาพ
 
วันที่สมัครสมาชิก: 21 กุมภาพันธ์ 2011
ข้อความ: 1,234
จูกัดเหลียง is on a distinguished road
Default

เรื่อง Holder ต้องรอผู้รู้ก่อนละครับ
เเต่ผมทำอย่างนี้นะครับ จากที่ $xyz=1$
พิจารณา จากอสมการ A.M.-G.M. $x+y+z\ge 3$ $$\Rightarrow \frac{1}{3}(x+y+z)^2\ge x+y+z$$
เเละ จากอสมการของ Chebyshev's $$x^2+y^2+z^2\ge \frac{1}{3}(x+y+z)^2$$
ทำให้ได้ว่า $$x+y+z\leq x^2+y^2+z^2$$


ปล. น้อง Black Dragon คงหมายถึงว่า ใช้ $x$ จำนวน $\frac{4}{3}$ ตัว ใช้ $y$ $\frac{1}{3}$ ตัว $z$ $\frac{1}{3}$ ตัว
จึงมีรวม $2$ ตัว เลยต้องไปคูณทางขวามันก็ โอเคนะครับ
เเต่ว่า มันนับไม่ได้ล่ะมั้ง( ที่ผมถามอ่ะ ไม่มีคนตอบเลย )
__________________
Vouloir c'est pouvoir

23 กรกฎาคม 2011 17:57 : ข้อความนี้ถูกแก้ไขแล้ว 3 ครั้ง, ครั้งล่าสุดโดยคุณ จูกัดเหลียง
ตอบพร้อมอ้างอิงข้อความนี้
ตั้งหัวข้อใหม่ Reply



กฎการส่งข้อความ
คุณ ไม่สามารถ ตั้งหัวข้อใหม่ได้
คุณ ไม่สามารถ ตอบหัวข้อได้
คุณ ไม่สามารถ แนบไฟล์และเอกสารได้
คุณ ไม่สามารถ แก้ไขข้อความของคุณเองได้

vB code is On
Smilies are On
[IMG] code is On
HTML code is Off
ทางลัดสู่ห้อง


เวลาที่แสดงทั้งหมด เป็นเวลาที่ประเทศไทย (GMT +7) ขณะนี้เป็นเวลา 20:44


Powered by vBulletin® Copyright ©2000 - 2024, Jelsoft Enterprises Ltd.
Modified by Jetsada Karnpracha